= les fonction dérivé determiner un maximum


  • B

    Bonjour à tous,
    je me permet de vous écrire afin que vous m'aidiez car j'ai beau réfléchir je ne trouve pas la solution et meme le debut du probleme ce qui savere embettant pour le reste du probleme...

    voils j'ecris lénoncé
    Un marchand de frites confectionne ses cornets de frites de la maniere suivante: il découpe un disque de papier de rayon Rqu'il entaille selon un rayon; il enroule ensuite le paier sur lui meme pour former un cone en recouvrant un secteur angulaire. Amateur de mathématiques il se demande comment confectioner un cone de volume maximael.

    on note:
    2pi -x l'angle de recouvremen t
    h la hauteur du cone
    r le rayon de labase

    rappel le voulme du cone est donné par la formule :V=pi r2h/3

    1)Préciser l'ensemble des valeurs possibles de h pour que que le probleme ait un sens

    1. exprimer r2 en fonction de h2 et de R2, puis le volume V du cone en fonction de h.

    et les questions d'apres se font grace a la fonction V trouvé précedemment le probleme je ne trouve pas!!!

    je vous remercie d'avance pour votre aide!
    bonne soirée
    babss!


  • B

    personne peut m'aider ???????je vous en supplieeeeeeeeeeee zoro je suis sure que toi tu peux y arriver!!! (lol)


  • B

    a nn meme toi ça me deçoit ce forum pour NUL si il y a un probleme un peu plus compliqué par raport a d'habitude sayé il n'y a pu personne moi je dis BRAVO!c'est sur c 'est pas un petit calcul et je vous ai mi que les premieres questions je reve!
    pff c nul


  • kanial
    Modérateurs

    Salut babss,

    D'une, il est toujours bon de rappeler que les gens qui répondent sur ce forum sont des bénévoles et que par conséquent ils ont d'autres activités que de passer leurs journées entières à y aider des élèves, il peut donc être bon de faire preuve de patience.

    De deux, tu peux toujours considérer que certains autres élèves qui postent sur ce forum sont nuls, c'est ton droit, mais sache que dans ce cas d'autres élèves postant sur ce même forum pourraient te considérer (si ils avaient la même mentalité) comme une vraie chèvre car l'exercice que tu postes est très loin d'être le plus difficile que l'on ait vu (tu devrais faire un tour dans les forums TS et supérieur...). En fait avoir de bons résultats en maths peut être une bonne raison d'aider les autres mais surement pas de les prendre de haut. Dis-toi bien qu'il y aura toujours plus fort que toi et qu'un peu d'humilité ne fait pas de mal.

    De trois, la critique est facile c'est sûr, quand on ne s'implique pas.

    L'aide pour ton exo ne viendra pas de moi, à bon entendeur...


  • B

    je suis tout a fait d'accord !mais comment expliquez vous que tous les autre exercices vous les resolvez et les autres qui ont un texte un peu plus long vous ne daignez jeter un coup d'oeil!

    bref veuillez m'excusez pour le désagrement causé mais j'étais un peu enervé et j'aurai aimé que vous m'aidiez en m'enervant mais bon apparamment je me suis bien trompée!
    pour information je ne suis pas du tout forte en maths par conséquant je ne prend personne de haut ici c'etait juste pour essayer d'attirer l'attention d'un correcteur ce qui aurait pu marcher ... mais bon je suis désolée les exos de maths supérieur bien sur sont plus difficiles mais ils ne sont pas de notre niveau et il y a des profs pour chaque niveau des profs plus fort qui resolvent les plus difficiles c'est normal
    enf1 bref je sais qu'il est pas forcément difficile (pour moi si ) mais il demande du temps que les bénévoles ne peuvent pas consacrer ,je comprends


  • W

    Relax max, cool raoul, tout vient à point à qui sait attendre,

    Le problème a un sens si h appartient à l'ensemble des réels positifs non nuls privé de +∞.
    Théorème de pithagore :
    r^2+h^2=R^2 ⇔ r^2=R^2-h^2
    Clairement, V=(pi*(R^2-h^2)h)/3 ⇔V=(pihR^2)/3 - (pih^3)/3.
    On dérive par rapport à h le volume, on obtient :
    (piR^2)/3 - (pih^2)=0 pour maximiser le volume.
    et : h^2=(R^2)/3
    Conclusion : pour maximiser le volume la hauteur h=+√((R^2)/3) puisque h>0.

    Tcho


  • W

    Une fois que tu as h et en supposant R connu, tu peux calculer l'angle :
    cos(alpha/2)=h/R ⇒ alpha/2=arccos(h/R) ⇒ alpha=2arccos(h/R)
    L'angle qui maximise le volume est alors de 2arccos(1/√3).


  • kanial
    Modérateurs

    désolé j'y suis peut-être allé un peu fort, mais il faut bien comprendre que le problème ne vient pas de la difficulté ou de la longueur de l'exo mais surtout du nombre d'exercices postés en cette fin de vacances (record de nombre de visites hier, alors qu'au début des vacances il se passait pas grand chose sur ce forum).
    Enfin évite quand même à l'avenir ce type de message assez désagréable et on se fera une joie de t'aider.
    Bonne reprise...


  • B

    une question est ce que l'on peut etudier le signe de la dérivée ou juste puisque l'on sait qu'il faut determiner un maximum on le sait que V doit etre croissante puis décroissante ?

    d'autre part, pour la dérivée de V je ne comprend pas pourqoi vous enlevez le h dans le 1er membre V=pihR^2/3 - pi *h^3 / 3
    V'=pi hR^2 - pi *h^2 non?

    moi j'avais trouvé h=R+r/2 ce qui me semblait plausible mais bon jai du me tromper qqpart

    merci enormément je vous suis entierement reconnaissante 😄


  • B

    oui j'éviterai non c'est moi qui suis désolée!! oui tous les devoirs maison c'est difficile de gérer tout ça!
    je comprend très bien ! mais vous etes bénévoles mais vous faites quoi comme métier ?


  • kanial
    Modérateurs

    Oui c'est vrai qu'il est mieux d'étudier le signe de la dérivée pour montrer qu'il s'agit bien d'un maximum mais ça devrait pouvoir se faire sans trop de difficultés vu l'expression de la dérivée : V'=(piR^2)/3 - (pih^2).

    Quant à cette dérivée justement, on dérive V=(pihR^2)/3 - (pih^3)/3 par rapport à h, pour mieux voir si tu veux tu peux appeler f la fonction définie par f(x)=[(piR^2)/3]*x - (pi/3)*x^3, on voit alors mieux que quand on dérive f par rappor à x, le x du premier terme disparaît et on retombe bien sur le résultat que t'as donné wiwiwi.

    La plupart de ceux qui répondent sont profs ou étudiant, pour ma part je suis étudiant (en maths spé actuellement...).


  • W

    Alors :
    c une constante
    x une variable
    la dérivée de cx, c'est c, Ok?. Donc la dérivée de ((piR^2)/3)h, c'est (piR^2)/3 .

    Conseil : toujours essayer de se servir de la ou des questions précédentes dans un problème de math.
    La première question étant d'exprimer r^2 avec les autres paramètres, il faut le remplacer dans la question suivante. Je ne vois pas comment tu peux avoir h=R+r/2 mais bon.
    Tu remarqueras que procéder ainsi permet d'obtenir l'angle qui est constant et ne dépend pas des paramètres (ce qui est assez joli).

    Tcho


  • B

    ah sayait j'ai compri pourqoi on enlevait le h!

    un enorme merci à cosmos et wiwiwi!!!!

    j'espere que j'aurais, grace a vous, une bonne note
    bonne continuation
    et peut etre a bientot.


  • B

    oui oui j'ai compris! je mettais servis de pythagore mais moi j'avais pas enlever le h et j'avais mis tout sur 3 (je n'ai pas l'impresion qu'on ait le droit)

    et quand j'ai mi ma dérivée egal a 0 pour trouver le maximum beaucoup de choses se simplifiait et je suis arrivée a R+r/2 mais bon je sais maintenant surrmeent pourqoi je me suis trompée!


Se connecter pour répondre